You are on page 1of 75

www.byjusexamprep.

com

Mock Test Solutions in English

Questions

1. Direction: Read the poem carefully and answer the following questions:

A slumber did my spirit seal;

I had no human fears:

She seemed a thing that could not feel

The touch of earthly years.

No motion has she now, no force;

She neither hears nor sees;

Rolled round in earth's diurnal course,

With rocks, and stones, and trees.

Why does the ‘I’ in the poem feel no human fear?


A. Because he thought that life would take its own B. Because he was confident in the abilities of his
course. beloved.
C. Because he thought that the passing of time
D. Because he was in a state of slumber.
would not affect her.
E. Because he was not aware of the effects of the
passing of time.
2. What could have happened to the SHE described in the poem?
A. She has become old and feeble. B. She has attained a state of salvation.
C. She has forgotten the poet and all that she left
D. She has lost her memory .
behind.
E. She has left for her heavenly abode.
3. Direction: Read the excerpt from the poem below and answer the questions that follow.
www.byjusexamprep.com

Love set you going like a fat gold watch.

The midwife slapped your footsoles, and your bald cry

Took its place among the elements.

Our voices echo, magnifying your arrival. New statue.

In a drafty museum, your nakedness

Shadows our safety. We stand round blankly as walls.

I'm no more your mother

Than the cloud that distills a mirror to reflect its own slow

Effacement at the wind's hand.

All night your moth-breath

Flickers among the flat pink roses. I wake to listen:

A far sea moves in my ear.

One cry, and I stumble from bed, cow-heavy and floral

In my Victorian nightgown.

Your mouth opens clean as a cat's. The window square

Whitens and swallows its dull stars. And now you try

Your handful of notes;

The clear vowels rise like balloons.

Which of the following is the theme of the poem?


A. A new mother’s emotions about motherhood. B. A child’s unconditional love for the mother.
C. The role of nature in a child’s life. D. The lack of family support for a new mother.
www.byjusexamprep.com

E. The role of creation and destruction in human life.


4. What are the rhetorical devices most used by the author?
A. Similes and antithesis B. Similes and metaphors
C. Alliterations and hyperbole D. Imagery and pathos
E. Synecdoche and metonymy
5. A new study published that all 77 species of birds surveyed by the study weigh less on average than they
did 40 years ago and nearly 80 percent of those species also have developed greater average wing-
lengths.Researchers aren’t yet sure what the consequences of these physiological changes might be or
the precise mechanisms that gave rise to them, but the team’s analyses suggest the rising temperatures
and changes in rainfall seen at the study site offer the most powerful statistical explanation for the birds’
transformation. The results fall short of demonstrating cause and effect, but show a strong association.The
study found that birds tended to be lighter following hotter and drier conditions than usual, especially if
those conditions fell during the dry season, which is the most stressful time of year for birds because food
is harder to find.

Which of the following statements can be BEST concluded from the passage?
B. Not all species of birds are affected by changes in
A. Plants and animals are affected by dry conditions.
weather conditions.
C. All bird species undergo bodily changes in 40 D. Statistical evidence indicates that climate change
years. is transforming the bodies of birds.
E. The last 40 years have witnessed more extreme
and destructive climate change events than ever
before.
6. Read the following statement carefully:

Vaibhav Purandare is neither a formal historian nor a/an __________. He is a journalist who has dared to
take up_________ historical figures to write books and throw new light on. What is particularly
adventurous is that he is neither intimidated nor _________ by the current ______and polarized political
atmosphere.

Fill in the blanks with the appropriate option:


A. Politician, staid, prohibited, vibrant B. Academician, incontrovertible, refrained, vicious
www.byjusexamprep.com

C. Scientist, controversial, inhibited, veracious D. Scholar, controversial, debilitated, prevaricating


E. Academician, controversial, inhibited, vicious
7. Replace the underlined part of the sentence with the option that is grammatically and logically suitable.

Apart from carbohydrates and vitamins, whole grains are a reliable source of proteins that are vital for
human health.
A. Apart from carbohydrates and vitamins, whole B. Besides carbohydrates and vitamins, whole grains
grains are a reliable source of are a reliable source of
C. In addition to carbohydrates and vitamins, whole D. Whole grains are a source of not only
grains are a reliable source of carbohydrates and vitamins but also
E. Other than carbohydrates and vitamins, whole
grains are a reliable source
8. Replace the underlined part of the sentence with the option that is grammatically and logically suitable.

Most of the stars are light years away; this means that the light coming from them took millions of years to
reach our eyes. As a result, we see them as they appeared during a period that is very far removed from
the time we know as now on earth.
A. we see them as they appeared during B. we see them as what they appeared like
D. we see them similarly to what they looked like
C. we see them as they were during
during
E. we see them as they happened to be during
9. A recent study finds that kids who played only individual sports, like tennis or gymnastics, tended to face
more mental health difficulties, such as anxiety and depression or troubles with concentration, than peers
who played no sports at all. Youngsters who participated exclusively in team sports, on the other hand,
were less likely to experience these issues than those who didn’t play any sport. Finally, the kids who
dabbled in team and individual sports were no more or less likely than those who abstained from sport
altogether to have mental health trouble.

Which of the following is likely to be true if the findings of the study are accurate?
A. Most kids prefer to play individual sports to team B. Most schools encourage team sports as they are
sports. less expensive.
C. The psychological benefits of playing sports have D. The physical benefits of playing sports offsets the
been overestimated. psychological drawbacks.
www.byjusexamprep.com

E. Coaches and parents are equally to be blamed for


the psychological pressures.
10. Arrange the following sentences in a LOGICAL sequence:

1). Buddhist monks in Thailand have established new ways to spread their religion that take advantage of
globalisation and religious tourism.

2). Not all Buddhist missionaries have to go out to find people ready to listen.

3). Thai monks created opportunities for non-Buddhist travelers to participate in a meditation retreat, join a
Buddhist community, talk with a monk, and they allow volunteers to teach English in a temple setting.

4). These cultural exchange programmes developed in the early 2000s from the confluence of education,
urbanisation and tourism in Chiang Mai, northern Thailand.

5). Those who are curious come to them.


A. 13452 B. 31425
C. 21543 D. 34152
E. 25134
11. Doctors stress the importance of hand hygiene: keeping your hands clean by regularly washing to prevent
the spread of infections.Unlike most other microbes, which can grow and reproduce on their own, viruses
must invade a host such as a human cell to reproduce. Because the virus seems to hang out in mucus
and other airway fluids, it spreads when infected individuals cough or sneeze. Released into the air,
infectious droplets can land on another person or a frequently touched surface like a doorknob, shopping
cart or through handshakes. After that, it’s a short trip for the virus from hand to head. Breaking this chain
of transmission can help stem the spread of disease

Which of the following can be most reasonably concluded from the argument?
A. There is no significant difference between a virus B. Someone carrying the virus should not sneeze or
and other microbes. cough.
C. Hand hygiene is simplistic and inconsequential D. Hand washing may be a simple yet effective
advice. method to curb the spread of a virus.
E. Hand sanitizer and soap manufacturers have
funded the study on hand hygiene.
www.byjusexamprep.com

12. White men hold most management positions in corporations, and women and minorities access fewer
leadership opportunities. The existence of this invisible barrier puts women at a disadvantage, all things
like competence and skills being equal. This is especially true for women of color because being a woman
in the first place has its consequences already. When pitted against white women, women of color are
usually considered inferior. It's not women's lack of talent, skill level, or ambition that prevents them from
accessing further opportunities. There are other factors at play, such as unconscious bias, that can affect
whether women move up the corporate ladder.

Which of the following, if true, would strengthen the author’s argument the most?
A. In general, women recognise great potential in B. In general, men recognise great performance in
other women. other men.
C. Women weren't as central to the workforce until D. There are more women in the workforce at the
recently. entry level.
E. It has been found that when a group of men
evaluate a woman's performance, they're less likely
to recognize her merits.
13. Direction: Read the passage carefully and answer the three questions that follow it.

One of the most fateful errors of our age is the belief that 'the problem of production' has been solved. Not
only is this belief firmly held by people remote from production and therefore professionally unacquainted
with the facts - it is also held by virtually all the experts, the captains of industry, the economic managers in
the governments of the world, the academic and not so-academic economists, not to mention the
economic journalists. They may disagree on many things, but they all agree that the problem of production
has been solved; that mankind has at last come of age. For the rich countries, they say, the most
important task now is 'education for leisure' and, for the poor countries, the 'transfer of technology'.

That things are not going as well as they ought to be going must be due to human wickedness. We must
therefore construct a political system so perfect that human wickedness disappears, and everybody
behaves well, no matter how much wickedness there may be in him or her. In fact, it is clearly known that
everybody is born good; if one turns into a criminal or an exploiter, this is the fault of 'the system'. No
doubt 'the system' is in many ways bad and must be changed. The main reasons why it is bad and why it
can still survive in spite of its badness, is this erroneous view that the 'problem of production' has been
solved. As this error pervades all present-day systems there is at present not much to choose between
them.
www.byjusexamprep.com

The arising of this error, so egregious and so firmly rooted, is closely connected with the philosophical, not
to say religious, changes during the last three or four centuries in man's attitude to nature. I should
perhaps say: western man's attitude to nature, but since the whole world is now in a process of
westernisation, the more generalised statement appears to be justified. Modern man does not experience
himself as a part of nature but as an outside force destined to dominate and conquer it. He even talks of a
battle with nature, forgetting that, if he won the battle, he would find himself on the losing side. Until quite
recently, the battle seemed to go well enough to give him the illusion of unlimited powers, but not so well
as to bring the possibility of total victory into view. This has now come into view, and many people, albeit
only a minority, are beginning to realise what this means for the continued existence of humanity.

What is the root cause of the fact that man, though born good, resorts to criminal behaviour?
A. People who are not professionally acquainted B. The fallacious belief that mankind has overcome
with the facts are calling all the shots. the problem of production.
C. The system is handled by people who do not D. Modern man is beset with the problem of being in
want to mend it. conflict with nature.
E. Politics of vested interests prevents the system
from mending itself.
14. Which of the following is not true according to the passage?
A. People believe that the main problem of B. All the systems that exist now suffer from a
production has been solved. common problem.
C. The powers that man believes that he has are not D. Man’s attitude towards nature has undergone sea
enough to give him complete victory over nature. changes in the last few centuries.
E. The western man, through his domination, seeks
to represent the whole of mankind.
15. What is the most likely outcome of the battle that man has waged against war?
A. A win-some, lose-some situation from the
B. Nature is unfathomable and, therefore,
perspective of mankind as he cannot hope for
unconquerable by anyone let alone man.
complete victory
C. Man can never win against nature because even D. Man enjoys unlimited powers and at least hopes
in his victory lies his own defeat. for partial victory in this battle.
E. Both nature and man would emerge as partial
winners and partial losers.
www.byjusexamprep.com

16. Arrange the following sentences in a logical sequence.

1). Proportionally large eyes, facing forward, and a round face.

2). It’s a characteristic you can identify in any animal that elicits a human response of ‘cute!!’.

3). Dogs look vulnerable, and in need of our care.

4). Much like a human baby.

5). Neoteny’ refers to the retention of juvenile traits in an adult both phenotypically and behaviourally.
A. 34521 B. 35421
C. 13245 D. 52143
E. 21534
17. Direction: Read the passage given below carefully, and answer the three questions that follow.

When I glance over my notes and records of the Sherlock Holmes cases between the years ’82 and ’90, I
am faced by so many which present strange and interesting features that it is no easy matter to know
which to choose and which to leave. Some, however, have already gained publicity through the papers,
and others have not offered a field for those peculiar qualities which my friend possessed in so high a
degree, and which it is the object of these papers to illustrate. Some, too, have baffled his analytical skill,
and would be, as narratives, beginnings without an ending, while others have been but partially cleared
up, and have their explanations founded rather upon conjecture and surmise than on that absolute logical
proof which was so dear to him. There is, however, one of these last which was so remarkable in its
details and so startling in its results that I am tempted to give some account of it, in spite of the fact that
there are points in connection with it which never have been, and probably never will be, entirely cleared
up.

It was in the latter days of September, and the equinoctial gales had set in with exceptional violence. All
day the wind had screamed and the rain had beaten against the windows, so that even here in the heart of
great, hand-made London we were forced to raise our minds for the instant from the routine of life, and to
recognize the presence of those great elemental forces which shriek at mankind through the bars of his
civilization, like untamed beasts in a cage. As evening drew in, the storm grew higher and louder, and the
wind cried and sobbed like a child in the chimney. Sherlock Holmes sat moodily at one side of the fireplace
cross-indexing his records of crime, while I at the other was deep in one of Clark Russell’s fine sea-stories,
www.byjusexamprep.com

until the howl of the gale from without seemed to blend with the text, and the splash of the rain to lengthen
out into the long swash of the sea waves. My wife was on a visit to her mother’s, and for a few days I was
a dweller once more in my old quarters at Baker Street.

Why does the writer want to give an account of one the cases despite the fact that some points regarding
it may never be clarified?
A. The case has been founded on conjecture rather B. The author found records of so many cases like
than logical ability which the author’s friend this that he did not know what to leave out and what
cherishes so much. to record.
D. The writer could not control his excitement at
C. The writer could not stop himself as this case had
finding a case that had no equals in terms of
astounding characteristics and astonishing results.
uniqueness.
E. The writer would not get another shot at recording
for posterity such a remarkable case.
18. Why was the writer staying with Sherlock Holmes?
A. The writer went there to retrieve his notes of the B. The writer was seeking refuge from the inclement
many cases involving his friend. weather.
C. The writer wished to reminisce about his old days
D. The writer’s wife was away visiting her mother.
as a bachelor in the absence of his wife.
E. The writer remembered his friend and wished to
stay with him.
19. Which of the following is not true as per the passage?
B. Some of the cases did not merit mention as they
A. The wind that particular day was unusual in its
did not have the scope of presenting the unique
intensity.
qualities the friend possessed.
C. Some of these cases may have baffled the
D. The writer was engrossed in reading some stories
writer’s friend but he always had the upper hand in
while his friend sat moodily across the fireplace.
the end.
E. The wind grew stronger as the day wore away.
20. Direction: Read the passage given below carefully and answer the three questions that follow it.

A writer in a well-known French newspaper once expressed his surprise: “Why is the notion of power
raised by so many people today? Is it such an important subject? Is it so independent that it can be
www.byjusexamprep.com

discussed without taking into account other problems?” This writer’s surprise amazes me. I feel skeptical
about the assumption that this question has been raised for the rst time in the twentieth century.

Anyway, for us it is not only a theoretical question but a part of our experience. I’d like to mention only two
“pathological forms” —those two “diseases of power”— fascism and Stalinism. One of the numerous
reasons why they are, for us, so puzzling is that in spite of their historical uniqueness they are not quite
original. They used and extended mechanisms already present in most other societies. More than that: in
spite of their own internal madness, they used to a large extent the ideas and the devices of our political
rationality.

Since Kant, the role of philosophy is to prevent reason from going beyond the limits of what is given in
experience; but from the moment since the development of the modern state and the political
management of society, the role of philosophy is also to keep watch over the excessive powers of political
rationality, which is a rather high expectation. Everybody is aware of such banal facts. But the fact that
they are banal does not mean they don’t exist. What we have to do with banal facts is to discover —or try
to discover— which specic and perhaps original problem is connected with them.

The relationship between rationalization and excesses of political power is evident. And we should not
need to wait for bureaucracy or concentration camps to recognize the existence of such relations. But the
problem is: What to do with such an evident fact? Shall we try to reason? To my mind, nothing would be
more sterile. First, because the eld has nothing to do with guilt or innocence. Second, because it is
senseless to refer to reason as the contrary entity to non-reason. Last, because such a trial would trap us
into playing the arbitrary and boring part of either the rationalist or the irrationalist.

I would suggest another way of investigating the links between rationalization and power. It may be wise
not to take as a whole the rationalization of society or of culture but to analyze such a process in several
elds, each with reference to a fundamental experience: madness, illness, death, crime, sexuality, and so
forth. I think that the word “rationalization” is dangerous. What we have to do is analyze specic
rationalities rather than always invoke the progress of rationalization in general.

Why is the author amazed at the surprise expressed by a well-known writer in a French newspaper?
B. Because the disease of power is not an original
A. Because the author is skeptical by nature.
idea.
C. Because questions on the notions of power have D. Because the well-known writer is asking this
been raised before. question for the first time.
www.byjusexamprep.com

E. Because the rationalization of power is something


that everybody has experienced.
21. Why does the author say that we should not use reason to examine the relation between rationalization
and excesses of political power?
A. Because we cannot decide what/who is guilty or B. Because reason and non-reason do not exist just
innocent using reason. as a binary.
D. Because this relation is beyond the scope of
C. Because the exercise would be very tedious.
reason.
E. Because reason is limited while this relation is
not.
22. Which of the following about the role of philosophy in human society is not true according to the passage?

i. The unattainable goal of monitoring the excessive powers of political rationality

ii. Excessive power may be historically unique but it is not original

iii. To keep reason within the limits of what is given in experience


A. All of i, ii, and iii B. Only ii and iii
C. Only i and ii D. Only ii
E. Only i
23. Direction: Read the passage carefully and answer the three questions that follow.

The word ‘Dharma’ has been misused in India since the past 1,500 to 2,000 years because many people
started to use Dharma in the sense of a religion or sect. However, Dharma is universal and eternal.
Dharma is not Buddhist, Christian, Hindu, Jain, Jewish, Muslim, etc. These are all different religions or
sects.

In ancient times, Dharma meant the universal laws of nature, which are applicable to everyone. Even
today, we come across Dharma being used in this sense. For example, in Hindi we say that it is the
Dharma of fire to burn and to cause to burn. Here, Dharma has nothing to do with any religion but means
the nature of fire. How can fire be Buddhist, Christian, Hindu, Muslim, Parsi or Sikh? Fire is fire. Likewise,
ice is ice. The Dharma of ice is to be cold and to cause to cool. This is the nature of ice, which is universal
and eternal. Similarly, when a person develops negativity such as aversion, jealousy, and animosity, he
feels unpleasant sensations and becomes miserable. Neither the mental negativity nor the resultant
www.byjusexamprep.com

misery can be called as Christian, Hindu, Buddhist or Jain. This is the nature of mental defilements: to
make one miserable.

When India became independent, the Indian government prepared its constitution, which stated that the
constitution and government would be Dharma-nirpeksh, meaning non-Dharmic. This was a blunder. How
can any government be non-Dharmic? It has to be Dharmic, meaning it has to give importance to right
conduct. Actually, the constitution should have stated that it is non-sectarian, not non-Dharmic. The
English word ‘secular’ was wrongly translated as Dharma-nirpeksh. Even today, national leaders and
others still use Dharma-nirpeksh.

Whenever a sectarian word is added to Dharma, it corrupts its meaning. The Buddha never added any
sectarian adjective to Dhamma. The only adjective that he added was ‘truth’. Therefore, he used the term
‘saddhamma’ (true Dharma) for his teaching. When the teaching is based on saddhamma, it does not give
rise to blind beliefs. The law of nature is the same for everybody. Therefore, the word saddhamma did not
give rise to a sect because the entire teaching is based on truth.

Why can’t a government be Dharma-nirpeksh according to the passage?


A. Because a government cannot be blind to the B. Because religion can never be separated from the
religious aspirations of its citizens. duties of a government.
C. Because a government will have to go according
D. Because a government cannot choose not to
to the laws of the land which are based on what is
heed what is right.
perceived to be right.
E. Because a government cannot avoid the path of
religious truth.
24. Which of the following cannot be described as Dharma according to the definition of Dharma given in the
passage?
A. Following the teachings of the Buddha B. Feeling hatred for a person who has hurt you
C. A tree giving oxygen that we breathe D. Following the precepts of a pagan religion
E. Water quenching the thirst of living beings
25. Which of the following would mitigate the claim(s) made in the passage?

i. The Dharma has always been misused from ancient times till now.

ii. The word ‘secular’ has been translated as Dharma Nirpeksh.


www.byjusexamprep.com

iii. Religious sects are different from the concept of Dharma.


A. Only i B. Only ii
C. Both i and ii D. Both ii and iii
E. None of i,ii and iii
26. Which of the following sentences do not contain errors?

i. Toxic waste should be handled and disposed carefully.

ii. Many people are indisposed to attend this session.

iii. He did not apprise me about the recent developments.

iv. Aim for the moon and you may well reach the peak of a mountain.

v. Please fill up this form at the earliest.

vi. He really looks up to his coach.


A. i, iv, v B. i, iii, v, vi
C. iv, v, vi D. ii, v, vi
E. ii, iv, vi
27. Harish is the branch head of a mid-sized banking and insurance company named ICU. Raka, the chief of
the accounts department of this branch, retired three months ago after putting in a valuable 30 years of
service. He was an exemplary employee, and everyone respected him. Harish hired Roki, a highly
qualified but less experienced (compared to Raka) professional. Roki is an alumnus of the reputed London
School of Banking, and he joined a month after Raka retired. There are three senior managers who report
to Roki. They have been in the organisation for around 12 years and got along very well with their earlier
boss.

Roki seeks an appointment with Harish and informs him that the three senior managers are proving to be
a nuisance. According to him, they do their best to not cooperate with him and even question everything
that he tells them to do. They always cut in when he is speaking. Roki feels they are trying to undermine
his leadership.

Which of the following should be the immediate response by Harish?


A. Ask Roki to send the three senior managers to B. Set up a meeting with all the four of them and
www.byjusexamprep.com

meet him in his office and find out what the problem settle their issues.
is.
C. Ask the HR manager to get to the root of the D. Tell Roki to issue show-cause notices to all the
issue and report to him. three of them.
E. Advise Roki to be mature enough to handle his
own problems.
28. Harish receives input from Raka that the three senior managers have opened up to Raka. They told Raka
that Roki just has a fancy degree and lacks practical knowledge. He supposedly needs to be subtly guided
many times. Everybody in the department, the three claim, knows about it but are keeping mum to avoid
unwanted trouble.

Which of the following would best prove the claims of the three to be invalid?
B. The three senior managers approached the HR
A. Roki’s record in his previous job is impeccable
Manager and told him that someone from the
and he came with a recommendation from a person
department should have been promoted instead of
that Harish knew personally.
hiring Roki.
C. One of the three has been subjected to an inquiry D. There has been a complaint last year from a staff
of trying to commit forgery but was let off for want of member that these three expected him to work
evidence. overtime without proper cause.
E. The people who work on the floor below the
accounts department have complained that these
three often smoke in the staircase.
29. Harish’s secretary knows the three senior managers very well. She tells Harish that they are very good
friends and were classmates in college. In her opinion, the accounts department is functional after Raka
retired because of the three senior managers. They are highly skilled and committed.

Which of the following will lend the most credence to the secretary’s claims?
A. Roki was on leave for 25 days as he had to
B. A lot of employees say the same thing as the
undergo a surgery and during that period, everything
secretary.
went on smoothly.
C. The best employee award has been won by one
D. Raka is also of the same belief.
of these three for the last six awards.
E. The HR department has confirmed that there has
www.byjusexamprep.com

never been any complaint against the three.


30. Ramesh is a senior programmer in Pretech Pvt. Ltd. He has been a reliable employee. He has put in 10
years of work and was expecting a promotion last year. Because of the pandemic, all employees’
promotions and hikes had been put on hold last year. Pretech has announced that all pending
promotions/hikes will be considered in the upcoming appraisal cycle, which is due next month. Ramesh
knows that the HR head is meeting the top management next week to discuss promotions. It is the job of
the HR Head to recommend any promotion.

At 2 p.m. on a Wednesday, Ramesh receives a request through email from the HR head. This is the first
time that the HR head has contacted Ramesh personally. The request is to send some gift vouchers to the
clients of Ramesh’s department. The gift vouchers cost Rs. 10000 each, and there are 200 of them to be
sent. The vouchers are all ready and have to be just picked up from the HR department. Ramesh is
authorised to pick up the vouchers and send them. The email also says that the HR head will be busy in a
meeting when the vouchers are supposed to be sent.

Ramesh is quite happy to comply with this request and he does the needful.

Which of the following statements, if true, is most likely to make Ramesh hopeful that he may be
promoted?
A. Everybody has received a similar request of help B. The HR Head is known to believe in the policy of
form the HR Head in the last three months. give and take.
C. This is not the first time Ramesh has been D. The HR Head chose Ramesh over others for
approached by the HR Head. help, while she is busy in a meeting.
E. Ramesh’s immediate junior, Dhera, who agreed to
whatever request the HR Head had, was promoted
last year.
31. Ramesh does not hear from the HR head for the next few days. Which of the following is the most
reasonable step that Ramesh can take next?
A. Approach the HR Head during lunch and enquire B. Send a polite email to the HR Head enquiring
if he is going to be recommended for promotion. about his future in the organisation.
C. Do nothing as the appraisal is due next month
D. Try to find out who is close to the HR Head and
and he will get to know about everything in due
ask that person about his promotion.
course of time.
www.byjusexamprep.com

E. Enquire if all went well in the delivery of the gift


vouchers and all the clients have received them.
32. A month afterwards, the appraisal takes place and Ramesh’s promotion has been put on hold for another
year. The HR Head cites Ramesh’s mediocre performance as the reason for the same. Which of the
following is the most reasonable step for Ramesh?
A. Blame the HR Head, who has not been B. He should look for ways to shore up his own
forthcoming with the expectations of Ramesh. performance.
C. Ramesh’s luck is to be blamed as it cannot be D. Ramesh should approach the higher
coincidence that he has been neglected for management and tell them that he has been
promotion two times already. deliberately neglected.
E. Ramesh should look for a new opportunity
elsewhere.
33. RKS is a super celebrity with millions of fans across the country. His image is that of a self-made, no-
nonsense but highly patriotic person. RKS has been the brand ambassador of POLA (a soft drink giant
based in the US) for the past 20 years.

Recently, a representative of an NGO came on national television with a study that showed that the
groundwater in and around the area where POLA’s main plant is situated has arsenic levels higher than
the permissible level. In the same programme, he also exhorts everybody to avoid Pola soft drinks as
much as possible.

Soon after this incident, RKS received a backlash on social media. Like what generally happens on social
media, many people are making memes and trolling him for issues that have hardly anything to do with
the suspected arsenic in the POLA incident. Some have even resorted to blaming him for his religion.

What would be the advice that RKS’s PR consultant gives to him in this regard?
A. Go to the cyber-crime department and ask for B. Hire a hacker and give a fitting reply to all the
action to be taken against his detractors. trollers.
C. Conduct a press conference and declare that he
D. Wait out for this wave to subside.
is not going to be associated with POLA any longer.
E. Stop signing any endorsement deals with any
corporate houses.
34. RKS receives summons from a local court in a small town based on a complaint filed by a citizen. The
www.byjusexamprep.com

citizen says that RKS should be taken to task as he endorses POLA and is, therefore, responsible for any
harmful effects of consuming POLA beverages.

Which of the following is the most apt response by RKS?


A. File a counter complaint as he is not directly B. Reach out to the man personally and ask him to
involved in the production of the soft drinks. withdraw his complaint.
C. Respond to the summons and keep ready the D. Release a statement in the press along with a
latest report of the Food Safety and Standards government authorised agency’s report saying there
Authority of India exonerating POLA is nothing wrong with POLA
E. Respond to the summons and seek immunity
from any unfair legal action.
35. A government authorised study finds that the claims made in the TV programme are correct. There is
credible information that the govt is likely to conduct an enquiry on POLA.

Which of the following should be the immediate step taken by the stakeholders of Pola?
A. Prepare a legal strategy to soften the blow: B. Ask the legal team to prepare a response on the
arsenic is deadly and there is no going out of this grounds that arsenic in groundwater is not Pola’s
unscathed. responsibility.
D. Conduct an internal enquiry to find out if there is
C. Flee the country as there is no way out. arsenic in beverages made in the plant mentioned
above.
E. Hire the best law firm in the country to defend
itself.
36. Portolocus is a transportation company with pan-India operations and a strength of 1000 employees. Jabir
is the HR manager in its Delhi office. One day, Jabir hears on the grapevine that an employee, Mayank, in
the purchase department is planning to resign soon as his spouse has switched and moved to a company
in Bengaluru. Mayank has been working with Portolocus for 3 years now, and there were recently many
concerns about his performance within the purchase department. Jabir stepped in at that time and helped
in resolving those concerns.

On hearing this piece of information about Mayank, what should Jabir do?
A. Not do anything as he has not received this news B. Not do anything as there have already been many
from any official communication channel. concerns about his performance and it will be good if
www.byjusexamprep.com

he resigns.
D. Confront Mayank about the situation so that the
C. Try to find more about the situation through the
truth about his spouse’s transfer and his plans is
grapevine channel.
revealed.
E. Discuss the matter with Mayank’s manager to
give her a heads-up and let her find out more about
the matter.
37. During the performance appraisal process, the issue of the employee’s plans comes up and Jabir subtly
leads Mayank to reveal that while his spouse has indeed moved to Bengaluru, he has no immediate plans
of moving with her. However, Mayank’s facial expressions and unsure tone leave Jabir unconvinced.

Based on this revelation, which of the following actions should Jabir take?
A. Make Mayank sign a bond with the company at B. Postpone Mayank’s increment by three months to
least for the next one year. see his seriousness towards staying.
C. Based on his assessment of Mayank’s D. Recommend him for an increment commensurate
expressions, give him a low rating on the parameter with his previous year’s performance, but nothing
of his commitment towards the organisation. more.
E. Find out more about Mayank’s true intentions
from colleagues close to him.
38. Three months later, Mayank resigned. He openly cites discrimination by his category manager as the
reason for his resignation. Jabir is asked by the CEO to enquire into this matter and submit a report in 2
days, giving his recommendations. He finds out that Mayank had enquired with the HR Department about
possibilities for a transfer to the Bengaluru branch but had been turned down because of a lack of vacancy
in that branch.

Which of the following should Jabir state in his report?


A. The category manager is vindicated as the reason B. As there were issues with Mayank’s recent
behind Mayank’s resignation is his desire to move to performance, it is possible that the manager’s
Bengaluru, so his allegations should be officially discrimination is tied to it. So, there is a need to take
refuted. some remedial action.
C. As the manager might have gotten wind of D. As the manager unfairly penalised Mayank for
Mayank’s plans on the grapevine, he might have some temporary performance issues, strict actions
related his performance issues with them and should be taken against him to set a precedent.
www.byjusexamprep.com

discriminated against him. So, the organisation


needs to be considerate with the category manager.
E. As clarity is needed on Mayank’s motivation
behind resignation, conduct detailed interviews, both
separately and together, with Mayank and the
category manager to make an informed decision.
39. You are a Human Resource (HR) Manager of a software firm. Your job is to conduct recruitment and
training projects, to initiate disciplinary investigations and to keep personnel records of staff. Your boss is
Kripa, one of the three senior managers, all of whom report to the Director HR, Drona. Any bill or budget
specifications have to be approved by the Director first.

Two internal candidates, Mr. Suresh and Mr. Ramesh, were interviewed to fill up the post of Senior
Software programmer. Mr. Suresh was selected as he performed better in the interview, had better
qualifications, and displayed more leadership qualities. Some days later, Mr. Ramesh met you and
complained that the selection was unfair as he had more experience than Mr. Suresh.

Which of the following is the best way to handle the issue raised by Ramesh?
A. Explain it to Mr. Ramesh that the right person for B. Explain to him that nothing can be done now as
the job is not always the one who has been around the process is over and if he wants he can contact
the longest. Drona.
D. Inform Ramesh that this complaint is
C. Allow Ramesh to vent out his anger and get done
unacceptable and may be held against him in the
with it.
next appraisal cycle.
E. Assure Ramesh that the process was fair, and if
he wants, you can share the relevant assessment
results and details.
40. Kripa has instructed you to organise a one-day induction programme at any nearby resort. You have 5 HR
executives to help you out and they are your reportees. The induction programme is a long-held tradition
in this firm.

Review the following steps:

i. Ascertain what is the budget quickly and allocate assignments to the HR executives.
www.byjusexamprep.com

ii. Send out emails containing the schedule of the programme.

iii. Zero in on a suitable resort according to the budget.

iv. Arrange transportation to and from the venue.

Arrange the above steps in descending order of urgency and relevance.


A. ii, iv, i, iii B. iii, ii, iv, i
C. i, iii, iv, ii D. iii, ii, iv
E. i, iii, ii, iv
41. You have received 3000 applications for the post of Trainee programmers (10 vacancies in all). You
suspect that one of the applicants is the son of the HR Director. Your appraisal is due and you are
expecting a promotion and all promotions take place through the Director. It is your job to fill up these
vacancies.

Review the following steps:

i. Approach the best people in the firm to conduct the selection process.

ii. Confirm if your suspicion is true.

iii. Conduct an onsite preliminary test to reduce the number of initial applicants.

iv. Organise group discussions as a part of the selection procedure.

Choose the steps to be followed and arrange them in descending order of urgency and relevance with
regard to the purpose of filling up these vacancies.
A. ii, iii, iv, i B. ii, iii, i, iv
C. i, iii, iv D. iii, iv, i
E. iii, i, iv, ii
42. Freebit Educational Group wants to grow by providing unmatched quality educational experience to its
students. However, the management can't afford training for enhancing its teachers’ skill. As an in-house
measure, the managing board thought of a scheme—teachers with high student ratings to mentor those
with less. Most of the teachers disliked this measure as they found that this system interfered with the
academic autonomy of teachers. Moreover, some junior teachers had higher ratings than senior teachers.
www.byjusexamprep.com

Many of the teachers are planning to quit and Treebit (a competitor) is even planning to woo away many
teachers.

Consider the following actions:

i. Call a meeting with representatives of the teacher-group who are planning to quit.

ii. Identify the group of teachers who are planning to quit.

iii. Look at the financial implications if some of the teachers will have to be retained at a higher salary.

iv. Run a brainstorming session among the leading teachers in the industry.

Choose the steps to be taken by the management of Freebit Educational Group, and arrange the steps in
descending order of relevance and efficacy in stopping the teachers from going to the competitors.
A. iv, i, ii, iii B. ii, iii, i
C. ii, iii, iv, i D. ii, iii, iv
E. ii, i, iii
43. Which of the following decisions is most likely to satisfy all the stakeholders?
A. Increase the salary of all deserving teachers after B. Think of an alternative way of improving teacher
a proper appraisal. skill.
C. Hire a teacher-trainer who will train all the D. Push ahead with the plans as those who want to
teachers. leave will in any way leave.
E. Allow teachers to supplement their income by
moonlighting with competitors.
44. You are a teacher in Freebit, and you have one of the highest teacher ratings in your field. The in-house
measure of higher rated teachers mentoring ones with lower ratings has kicked off. You have just been
assigned the task of mentoring three teachers. One of them has 5 years more experience than you.

Which of the following steps would ensure that everything runs smoothly?
A. Request the management to assign another B. Call a meeting and let all know how things are
mentor to the senior teacher. going to be going forward.
C. Meet the senior teacher and assure him that you D. Get in touch with the three teachers and convey
will not be causing any problems for him. that everybody is going to learn from one another.
www.byjusexamprep.com

E. Meet all the three and discuss what they feel


about the whole issue.
45. Abani owns Abani & Sons Pvt. Ltd., the biggest business conglomerate in the country. Rekha, Sukhesh,
and Sunil are his children. Rekha was the eldest sibling and Sunil the youngest. Mrs. Abani passed away
when Sunil was just a toddler. Rekha was a motherly figure to her two younger siblings. The two elder
siblings joined the family business. Mr. Abani wanted people who he trusted to hold the key positions in
his conglomerate and was eagerly waiting for Sunil to join his other siblings in the family business. Mr.
Abani wanted Sunil to pursue business studies as he felt that Sunil had the most business acumen among
his children. However, Sunil wanted to be in the field of Mass Communication. In fact, very recently he got
a seat in the best institute for Mass Comm in the country. Sunil wished to share the news of his admission
into the best institute of Mass Comm at the earliest.

Sukhesh was the first to get this news from his brother. He was happy that Sunil got what he wanted, but
he was also worried about their father’s reaction. Mr. Abani had a nervous breakdown a few weeks ago.
He recovered but was resting in their farmhouse.

Which of the following, if true, will present the BEST opportunity for Sunil to break the news to his father at
the earliest?
A. Abani took a 45-minute walk every day as a part
B. Abani’s birthday, which is a month away
of the recuperation process.
C. A calm evening with Abani once he recovers D. Sukhesh asked Sunil to break the news to Abani
completely as soon as possible.
E. Writing a letter to Abani as Sunil had this habit of
writing letters to his father since childhood.
46. Abani has directed Sukhesh to pick the head of a new manufacturing plant. It goes without saying that
every new venture or branch is moulded in lines with the ethos and working style of Abani & Sons Pvt. Ltd.
Sukhesh also wants to pick a candidate acceptable to Abani.

Which of the following candidates is the BEST choice for the position?
A. Sridevi (Sukhesh’ wife and an MBA by B. Shikha, the head of another plant that is just 50
qualification); she has been a wonderful homemaker km away from the new plant; she has handled her
and quite close to Abani. job exceptionally.
C. Sunil, who has a year left with him before joining D. Clara, the most well-known employee; she has
www.byjusexamprep.com

the college. received 3 employees of the year awards


consecutively.
E. Ravi, the deputy head of another plant; he has
been with Abani Sons for the last 30 years and is
due to retire in 8 years.
47. Rekha wanted to be an actress when she was young. Abani discouraged her from taking this career path
and made her join the family business. She used to be quite upset with her father’s decision back then,
but now, she has reconciled and sticks to her father’s ideals and values. Sukhesh is of the opinion that
Rekha would side with Abani and might try to dissuade Sunil from joining the college.

Which of the following, if true, will BEST mitigate Sukhesh’s apprehension regarding Rekha?
B. Rekha had earlier supported an inter-caste
A. Rekha still does part-time acting, taking up small
marriage of two employees despite Abani’s
roles whenever she can in her free time.
disapproval.
D. Rekha has written an autobiography in which she
C. Sunil, right from the time he was small, always
describes the episode of disagreement over career
had a mind of his own and stuck to his own decision.
choice with her father.
E. When Sunil was 15 years old, Rekha had allowed
Sunil to join a school that he wanted to go to despite
disagreement from Abani.
48. Bhanu started writing all the natural numbers in an order on a piece of paper. If he could write only 1200
digits, then what is the last complete number that he wrote on the paper?
A. 430 B. 432
C. 434 D. 436
E. None of the above
49. Two drivers start simultaneously from points X and Y and move towards each other. They travel with their
own uniform velocity. Having arrived at their destinations, they turned back to their starting points. They
met for the first time on their onward journey 20 km from Point Y. They met for the second time on their
return journey 12 km from Point X. Find the distance between Point X and Point Y.
A. 36 B. 40
C. 48 D. 52
E. 56
www.byjusexamprep.com

50. 400 students, who are studying one course amongst A, B, C, or D, are selected from a college for inter-
college games. Out of the total number of selected students, the percentages of the students who were
selected from courses A, B, C, and D are 20%, 25%, 15%, and 40%, respectively. Out of the students
selected from the given courses, the percentage of students who are athletes are 40%, 55%, 25%, and
50%, respectively. The number of students who were selected from courses A and B and are not athletes
is approximately what per cent of the number of students who are athletes selected from courses C and
D?
A. 94% B. 96%
C. 98% D. 100%
E. None of the above
51. A number when divided by 9, 8, 6, and 5 gives the remainder as 7, 6, 4, and 3, respectively. The greatest
such 4-digit number is:
A. 9828 B. 9718
C. 9928 D. 9678
E. None of the above
52. A student attempted five written tests, each of which had the same maximum marks. When the father of
the student asked him about his marks, he told him the average of each possible pair that can be formed
using the marks obtained by him. These averages were 77, 103, 110, 84, 91, 108, 89, 117, 115, and 122.
If he obtained distinct marks in each of the five tests, then find the second highest marks obtained by him.
A. 111 B. 112
C. 115 D. 120
E. 122
53. Direction: Based on the given information, solve the questions that follow.

Abhishek was planning to buy a laptop, and he decided to purchase one among Panto, Luka, Marty,
Gubera, and Jamiko. He compared the five laptops based on the five parameters: price, processor,
storage, RAM, and size. He rated all the five laptops on each of these parameters on a scale of 10 to 50,
each rating being a multiple of 5. He plotted a radar chart (as plotted below) based on his ratings, but he
deleted the legend of the chart that showed the name of the laptop brand by mistake.
www.byjusexamprep.com

He calculated two ratings using the above parameters as given below:

a. Performance rating for any laptop: The weighted average of the three parameters, processor,
storage, and RAM, with weightage as 0.4, 0.2, and 0.3, respectively.

b. Overall rating for any laptop :

He decided to buy the laptop which has the highest final rating. He used the following formula to
determine the final rating.

Final rating = Performance rating + 10 (overall rating)

However, the following information is known.


www.byjusexamprep.com

1. Performance ratings of Luka and Gubera were the same, but Luka had a better overall rating than
Gubera.

2. The overall rating of Marty was less than that of Panto, But its performance rating was better than
Jamiko.

Which of the following laptops would Abhishek purchase?


A. Panto B. Marty
C. Luka D. Gubera
E. Jamiko
54. What was the overall rating of Gubera?
A. 5 B. 4
C. 6 D. 3
E. None of the above
55. What is the absolute difference between the final rating of Gubera and Panto to the nearest integer?
A. 17 B. 19
C. 21 D. 29
E. 23
56. If , then for what value of x, f(x,7) is 67?
A. 0 B. –9
C. 2 D. Both B and C
E. None of the above
57. Which day of the week is 1 February 2013?
A. Sunday B. Monday
C. Tuesday D. Thursday
E. Friday
58. Let p = . Find the remainder when p is divided by 11
A. 7 B. 6
C. 5 D. 8
E. 9
59. In the figure given below, Triangle ABC is drawn such that AB = 10 cm, BC = 26 cm, and AC = 24 cm. AE
touches Side BC at Point E. The centroid G lies on Side AE. Find the length of GE in cm.
www.byjusexamprep.com

A. 3 B. 3.65
C. 4.33 D. 5.12
E. 5
60. Find the smallest value of ‘P’ such that the LCM of 68, 128, and P is 248.
A. B.
C. D.
E.
61. Direction: Based on the given information, solve the questions that follow.

In a tournament, there were two groups, A and B. In each of the two groups, there were four teams. The
teams in group A were Bakka, Chakka, Jakka, and Pakka, while teams in group B were Akka, Dakka,
Nakka, and Takka.

There were three rounds in the tournament: First round - Super 8, Second round - Super 4 and Third
round - Super 2.

The points are awarded in each round as per the following table:
www.byjusexamprep.com

In the first round, every team plays against every other team only once in each group. At the end of the
first round, the maximum difference between the points scored by any two teams in group A is 8, while
that in group B it is 24. After the first round, the top two teams of the two groups enter the second round.
The top team of each group plays with the team ranked second (based on points) of the other group. If the
top two teams have the same number of points after the first round, then a toss will be done to decide who
will play with which team of the other group in the second round.

The points gained/lost by any team in the first round is carried forward to the further rounds. The
maximum difference between the points scored by any two teams after the second round was 20, and the
two teams that won their second round encounters proceeded to the third round, and they were from
different groups. The team from group B won the tournament. No game ended in a draw in any stage of
the tournament.

What was the total number of points of the team from group A that lost in the final after the second round ?
A. 8 B. 10
C. 12 D. 16
E. 14
62. If Akka has won only against Nakka and Dakka has lost only one match in the first round, then who among
the following would have won the tournament?
A. Akka B. Nakka
C. Takka D. Dakka
E. Cannot be determined
63. If Chakka won three matches in the tournament and Bakka lost only against Chakka in the first round,
then what is the absolute difference between the net points of Chakka and Bakka at the end of the
tournament?
A. 0 B. 2
C. 4 D. 6
www.byjusexamprep.com

E. None of the above


64. On a 10 × 10 chessboard, Kishore selected 3 squares. Find the probability such that these squares are in
different rows and columns.
A. B.

C. D.

E.
65. If each vowel in the given options represents an odd number and if each consonant represents an even
number, then which of them is true?
A. b2 + a3 – 3be2 + xy is even. B. 3c2 + d5 – e2u is even.
C. 7a2 + 4a is odd. D. 8k2 – 11p is odd.
E. None of the above
66. Direction: Read the data given carefully, and answer the questions that follow it.

Five movies (The Bourne Identity, The Transporter, Predator, Lethal Weapon, and Gladiator) were being
shown on seven screens of Cinebuff multiplex everyday. Each movie was shown on exactly four different
screens everyday. The number of movies shown on a screen must be one of 2, 3 or 4. The number of
movies shown on screens 2, 3, 4, and 7 were equal on all days. The numbers of movies shown on
screens 1 and 6 were equal on all days. The following grid shows the combination of seven screens with
the five movies. Here, â indicates that a particular movie is not shown on that particular screen. The grid is
not completely filled.
www.byjusexamprep.com

1. Lethal Weapon and Gladiator were shown on two different screens out of Screen 1 and 6.

2. One out of The Bourne Identity and The Transporter was shown on Screen 4 and the other on Screen
3.

3. No movie is shown on more than two screens that have consecutive screen numbers.

The Bourne Identity was definitely shown on which of the following screens?
A. Screen 3 and screen 2 B. Screen 4 and screen 5
C. Screen 2 and screen 5 D. Screen 1 and screen 2
E. Cannot be determined
67. How many of the following statements are definitely correct?

(i) Predator was shown on Screen 3.

(ii) The Bourne Identity was shown on Screen 4.

(iii) Gladiator was shown on Screen 7.

(iv) Lethal Weapon was shown on Screen 6.


A. 1 B. 2
C. 3 D. 4
E. 0
www.byjusexamprep.com

68. For how many movies can the four screen numbers that they are shown on be uniquely determined?
A. 1 B. 2
C. 3 D. 4
E. 5
69. The HCF and LCM of two positive integers are 32 and 576, respectively. If each ordered pair of the
possible values of these integers is represented as (x, y), where x < y, then find the sum of all the values
of x.
A. 3 B. 10
C. 20 D. 27
E. None of the above
70. Two big circles, each of whose radius is (3 + 3 ) cm, and one small circle whose radius is 3 cm touch
each other such that the line joining their centres passes through their point of contact. Find the perimeter
(in cm) of the region bounded by these three circles.
A. B.

C. D.

E. None of the above


71. Directions: The question given below consists of a question and two statements that are numbered I and
II. Decide whether the data provided in the statements is sufficient to answer the question. Read both the
statements, and answer accordingly.

Find the number of coins of 25 paisa.

Statement I: In a bag, the ratio of the number of coins of 25 paise to that of 50 paise to that of Re. 1 is 3 :
4 : 5, respectively.

Statement II: The ratio of the difference between the value of Re. 1 coins and 50 paise coins and that of
50 paise coins and 25 paise coins is 12 : 5.
A. The data in statement I alone are sufficient to B. The data in statement II alone are sufficient to
answer the question, while the data in statement II answer the question, while the data in statement I
alone are not sufficient to answer the question. alone are not sufficient to answer the question.
C. The data in either statement I or statement II D. The data in both statements I and II together are
alone are sufficient to answer the question. necessary to answer the question.
www.byjusexamprep.com

E. The data in both statements I and II are not


sufficient to answer the question.
72. Direction: Read the data given below carefully, and answer the questions that follow it.

The following chart shows the data about the attendance of five members of parliaments (Amrit, Som,
Sudha, Amiya, and Piyush) in five sessions with 50 working days per session. The names of the members
of parliament are disguised as V, X, W, Y, Z in some order.

Some additional points that are known are given below.

1. Amiya’s attendance and Sudha’s attendance are equal in one session, which in turn is double of
Piyush’s attendance for one session and it happened in three different sessions for these three people.

2. Piyush’s attendance is equal to half of Sudha’s attendance for two different sessions on exactly two
occasions. Also, Piyush’s attendance is double of Amrit’s attendance in two different sessions on exactly
one occasion.

For how many sessions is the attendance of Som more than 80% in a session?
www.byjusexamprep.com

A. 1 B. 2
C. 3 D. 4
E. 5
73. For which of the Members of Parliament is the ratio of attendance in two sessions an integral multiple of
5?
A. Som B. Amrit
C. Piyush D. Sudha
E. Amiya
74. If the attendance of the five Members of Parliament are ranked in descending order for each session, then
the attendance of Amrit was never ranked.
A. 1st B. 2nd
C. 3rd D. 4th
E. More than one of the above
75. Solve for the number of integral values of x:

A. 5 B. 6
C. 7 D. 8
E. 3
www.byjusexamprep.com

Solutions

1. C
Sol. The answer is implied in these lines: ‘I had no human fears: She seemed a thing that could not feel The
touch of earthly years.’ The greatest human fear is the fear of death, and not feeling the touch of earthly
years means not affected by the passing of time. We also know that the passing of years brings us closer
to death.
2. E
Sol. From the second stanza, it is clear that she died. No motion, no force, not seeing or hearing all point
towards death. Option B goes a step beyond what is implied here as salvation is more of a spiritual thing
and is not synonymous with death.

Hence E is the correct answer


3. A
Sol. Poem summary:

The poem describes a new mother’s emotions about motherhood. The poet uses similes and metaphors
to describe the baby and the mother.

Option A is the main theme of the poem.

Option B is irrelevant as the poem does not mention the infant’s emotions.

Option C is incorrect. Though the poet uses words associated with nature to describe the baby and the
mother, the words are not used to indicate the role of nature in one’s life.

Options D and E are irrelevant.

Hence, option A is the answer.


4. B
Sol. The author uses similes like ‘like a fat gold watch’, ‘clean as a cat's’, ‘vowels rise like balloons’, etc.

He also uses metaphors like ‘new statue’, ‘a cloud’, etc.

Hence, option B is the correct answer.


www.byjusexamprep.com

5. D
Sol. The passage is about a study which shows that birds were lighter and had greater average
wing spans than they did 40 years ago. The author also states analyses suggesting that the
rising temperatures and changes in rainfall seen at the study site offer the most powerful
statistical explanation for the birds’ transformation.
From this, we can conclude that the rising temperatures and changes in rainfall, or the climate
changes, are responsible for the bodily changes.
Hence, option D is the correct answer.

6. E
Sol. In the first blank, ‘academician’ or ‘scholar’ are suitable. The person described here is writing on historical
figures despite not being a historian.
For the second blank, the verb ‘dared to’ is the trigger, and we expect something that is likely
to cause trouble or difficulty. Controversial is the right fit here. Staid means serious and boring.
Incontrovertible means something that cannot be denied or refuted.

For the third blank, inhibited, meaning to be kept in check, is the best choice to go along with
‘intimidated’ that occurs just before this blank. ‘Refrained’ is avoided and does not fit here.
‘Debilitated’ means weakened or crippled.

Vicious, meaning fierce or aggressive, goes perfectly well with the polarized atmosphere.
Vibrant and veracious (truthful) carry positive connotations and do not fit here.

7. D
Sol. In options A, B, and C, the part before the comma acts like a modifier and would modify the subject
(Whole grains). This indicates the wrong sense that carbohydrates and vitamins are sources of protein
(just as whole grains are). In option E, ‘other than’ conveys the sense of an exception and is unsuitable
here. Option D conveys the correct sense that whole grains are a source of carbohydrates, vitamins, and
proteins.
8. C
Sol. The sense here is that because of the fact that the stars are far away and the light from them took very
long to reach us, what we see now is what the stars actually were a long time ago. The word ‘appear’,
www.byjusexamprep.com

when used in this context, would be similar to the word ‘seem’ and is unsuitable here. This makes options
A and B wrong. In option D, ‘we see them similarly’ means that the way or manner of seeing is the same
(this is wrong). In option E, ‘happened to be’ indicates a chance occurrence. What the stars were is not a
chance occurrence.

9. C
Sol. Options A and B are out of scope of the passage as nothing has been stated about the
preference of children or schools.
Similarly, the passage only states the findings of the study and does not mention the role of
parents, coaches. So, options D and E cannot be inferred.
Option C is correct; the passage mentions that kids who play individual sports face more
mental health issues.
Also, refer to the concluding lines of the paragraph. Finally, the kids who dabbled in team and
individual sports were no more or less likely than those who abstained from sport altogether to
have mental health trouble.

From the concluding lines, we can infer that playing both kinds of sports did not make a child
less likely to have mental health issues than one who did not play a sport.
Thus, we can infer that there are no psychological benefits in playing sports, or in other words,
the psychological benefits associated with playing sports are overestimated.
Hence, option C is the correct answer.
10. E
Sol. The first apparent mandatory pair is 2-5. The ‘them’ in 5 refers to the Buddhist missionaries in
2. This pair is very general in scope and would serve as a good beginning. 1-3 is also a clue. 3
gives us examples of the new ways (mentioned in 1) that monks in Thailand have established
to spread religion by taking advantage of globalisation. 4 follows 3 as the ‘these cultural
programmes’ are mentioned in 4.

11. D
Sol.
www.byjusexamprep.com

The passage describes how a virus is transmitted and focuses on hands as the main source of
contact. Refer to the first line of the passage: ‘Doctors stress the importance of hand hygiene:
keeping your hands clean by regularly washing to prevent the spread of infections.’
So, from the information given in the first line and the mode of transmission of viruses, we can
infer that keeping hands clean can curb the spread of infections through a virus too.
Hence, option D is the correct answer.
12. E
Sol.

The passage is about the glass ceiling and the existence of factors such as unconscious bias
that can affect whether women move up the corporate ladder.

Options A and B do not show a bias against women. Options C and D are facts but do not
strengthen the argument on factors that affect women.

Option E confirms the bias against women. Hence, it is the correct answer.

13. B
Sol. Passage summary: There are many people who wrongly believe that man has overcome the problem of
production. All the systems that exist in the world now need fixing, and the root cause of the problem,
again, is this erroneous belief that the problem of production is over. This erroneous belief arose because
of man’s changing attitude towards nature. Man even wants to wage war against nature not realizing that
even if he wins, he would still end up on the losing side.

Genre: Miscellaneous

Number of words: 425

Type of question: Specific idea-based question

Solution: The answer to this question is indicated in the second paragraph where it is given that man is
born good but becomes criminal because the system is bad and needs to be changed. The system is bad,
www.byjusexamprep.com

in turn, because of the erroneous belief that the problem of production has been solved. Thus, this wrong
belief is the root cause, and option B is the best answer.

Option A is not relevant to this question as it tells us about one of the sets of people who holds this belief.
Options C and E are beyond the scope of the passage. Option D pertains to the idea of western man
explained in the third paragraph and not to this question.
14. E
Sol. Passage summary: There are many people who wrongly believe that man has overcome the problem of
production. All the systems that exist in the world now need fixing, and the root cause of the problem,
again, is this erroneous belief that the problem of production is over. This erroneous belief arose because
of man’s changing attitude towards nature. Man even wants to wage war against nature not realizing that
even if he wins, he would still end up on the losing side.

Genre: Miscellaneous

Number of words: 425

Type of question: Specific idea-based question

Solution: Option A can be inferred from this sentence from the first paragraph: ‘For the rich countries, they
say, the most important task now is 'education for leisure' and, for the poor countries the “transfer of
technology”.’ The ‘they’ mentioned here refers to the various sets of people who believe that the problem
of production is now solved.

Option B is also true. Refer to this line: ‘As this error pervades all present-day systems, there is at present
not much to choose between them.’

Option C can also be inferred from this sentence: ‘Until quite recently, the battle seemed to go well enough
to give him the illusion of unlimited powers, but not so well as to bring the possibility of total victory into
view.’ ‘Him’ refers to the whole of mankind, and the ‘battle’ here is man’s battle against nature.

Option D refers to this sentence: ‘…the philosophical, not to say religious, changes during the last three or
four centuries in man's attitude to nature.’

Option E is not accurate as mankind can be represented by the western man as everybody is now
becoming westernised. This does not mean that the western man seeks to represent through his
www.byjusexamprep.com

domination.
15. C
Sol.

Passage summary: There are many people who wrongly believe that man has overcome the problem of
production. All the systems that exist in the world now need fixing, and the root cause of the problem,
again, is this erroneous belief that the problem of production is over. This erroneous belief arose because
of man’s changing attitude towards nature. Man even wants to wage war against nature not realizing that
even if he wins, he would still end up on the losing side.

Genre: Miscellaneous

Number of words: 425

Type of question: Inference-based question

The answer can be inferred from this sentence from the last paragraph: ‘He even talks of a battle with
nature, forgetting that, if he won the battle, he would find himself on the losing side.’ Option C comes the
closest to this idea.
16. D
Sol.
Sentence 2 should come after 5. The clause ‘It’s a characteristic’ in 2 is the ‘Retention of juvenile traits’
mentioned in 5. 1 and 4 form a pair as 1 mentions a few characteristics and 4 equates them with a human
baby. 3 sums up teh entire paragraph. Hence, the correct answer is D.

17. C
Sol. Passage summary: The writer notices that there is a huge number of cases involving his friend between
the years ’82 and ’90. It is difficult to choose what to keep a record of and not. Of the many cases, there is
one account that the writer wants to present because of its remarkable and startling results. It is late
September. There is a storm outside, the writer is reading some stories, and his friend is on the other side
of the fireplace cross-indexing his records of crime.

Genre: Fiction

Number of words: 380


www.byjusexamprep.com

Type of question: Specific idea-based question

Option A is correct about this case, but it is not the reason for the writer’s desire to give an account of this
case. Option B is also another statement taken from a part of the passage that has no relevance to this
question. Option C can be inferred from this sentence: ‘There is, however, one of these last which was so
remarkable in its details and so startling in its results that I am tempted to give some account of it, in spite
of the fact that there are points in connection with it which never have been, and probably never will be,
entirely cleared up.’ Option D is farfetched as there is no mention of this case having no equals. Option E
is beyond the scope of the passage.
18. D
Sol.

Passage summary: The writer notices that there is a huge number of cases involving his friend between
the years ’82 and ’90. It is difficult to choose what to keep a record of and not. Of the many cases, there is
one account that the writer wants to present because of its remarkable and startling results. It is late
September. There is a storm outside, the writer is reading some stories, and his friend is on the other side
of the fireplace cross-indexing his records of crime.

Genre: Fiction.

Number of words: 380

Type of question: Direct/Data-based question

The answer to this question is given clearly in the last sentence of the passage: ‘My wife was on a visit to
her mother’s, and for a few days I was a dweller once more in my old quarters at Baker Street.’ Option D
looks simple, but this is the closest to the given fact. Options A and B are irrelevant. Option C contains an
extra element of the writer wishing to reminisce or talk about pleasant memories. Option E: there is no
mention or indication that the writer remembered his friend.
19. C
Sol.

Passage summary: The writer notices that there is a huge number of cases involving his friend between
the years ’82 and ’90. It is difficult to choose what to keep a record of and not. Of the many cases, there is
one account that the writer wants to present because of its remarkable and startling results. It is late
www.byjusexamprep.com

September. There is a storm outside, the writer is reading some stories, and his friend is on the other side
of the fireplace cross-indexing his records of crime.

Genre: Fiction

Number of words: 380

Type of question: Specific idea-based question

Option A follows clearly from this sentence: ‘It was in the latter days of September, and the equinoctial
gales had set in with exceptional violence.’

Option B can be inferred from this sentence: ‘Some, ….. and others have not offered a field for those
peculiar qualities which my friend possessed in so high a degree, and which it is the object of these
papers to illustrate.’ The ‘others’ here refer to a kind of case. The motive of the writer presenting these
cases was to highlight the peculiar qualities of his friend, and these cases did not involve these qualities.

Option C is only partially correct and is the answer. The second part that the friend always had the upper
hand is not true as many of these cases were only partially cleared and some did not have endings.

Option D is directly stated in this sentence: ‘Sherlock Holmes sat moodily at one side of the fireplace
cross-indexing his records of crime, while I at the other was deep in one of Clark Russell’s fine sea-
stories…’

Option E is also correct according to this sentence: ‘As evening drew in, the storm grew higher and
louder…’ The way wearing away means the day slowly turning into evening.
20. C
Sol. Passage summary: The author is amazed that a French writer thinks that people are raising the question
of the notions of power only now. In fact, this whole notion has been a part of experience. Two pathologies
of power Stalinism and Fascism are apt examples. They did not do anything new but just extended and
used mechanisms already present in our societies. The role of philosophy is not just to keep reason within
the limit of experience but also to keep a check on the excessive powers of political rationality. The relation
between rationalisation and excesses of political power is evident. We don’t need to be reminded of it. We
also cannot use reason to examine and understand this relation. A better way to examine this relation is
not to take as a whole the rationalization of society or of culture but to analyze such a process in several
elds, each with reference to a fundamental experience: madness, illness, death, crime, sexuality, etc.
www.byjusexamprep.com

Genre: Philosophy

Number of words: 451

Type of question: Inference-based question

Option A is too large in scope to answer this question. The question is about the author’s skepticism at
one specific instance.

Option B is a fact mentioned in the 3rd paragraph, but it is not the reason behind the author’s skepticism
mentioned in the first paragraph.

Option C can be inferred from the first paragraph, especially this sentence: ‘I feel skeptical about the
assumption that this question has been raised for the rst time in the twentieth century.’ The question that
is mentioned in this sentence is the one raised by the French writer and the question is why so many
people are raising questions about the notions of power.

Option D is misguided. Nowhere is it stated that the French writer is asking his/her question for the first
time. Rather the author is amazed at the assumption (behind the French writer’s surprise in the form of the
question why people are raising questions on the notion of power as if it is very important.) that questions
are being raised for the first time on the notions of power.

Option E is beyond the scope of the passage.


21. B
Sol.

Passage summary: The author is amazed that a French writer thinks that people are raising the question
of the notions of power only now. In fact, this whole notion has been a part of experience. Two pathologies
of power Stalinism and Fascism are apt examples. They did not do anything new but just extended and
used mechanisms already present in our societies. The role of philosophy is not just to keep reason within
the limit of experience but also to keep a check on the excessive powers of political rationality. The relation
between rationalisation and excesses of political power is evident. We don’t need to be reminded of it. We
also cannot use reason to examine and understand this relation. A better way to examine this relation is
not to take as a whole the rationalization of society or of culture but to analyze such a process in several
elds, each with reference to a fundamental experience: madness, illness, death, crime, sexuality, etc.
www.byjusexamprep.com

Genre: Philosophy

Number of words: 451

Type of question: Inference-based question

The author mentions three reasons for this in the fourth paragraph. Option A looks close to the first reason
but the first reason is that this field or line of enquiry has nothing to do with guilt or innocence. It is not
about the failure to decide who/what is guilty or innocent.

Option B is the second reason phrased in other terms. When we say that it is senseless to refer to reason
as the contrary entity to non-reason, it means reason and non-reason does not just exist as two opposite
poles.

Option C: what is described as boring in the third reason is the part of being a rationalist or a non-
rationalist. The attempt to examine the relation between rationalization and excesses of political power is
not what is described as boring.

Options D and E are not stated / cannot be inferred from anywhere in the passage.
22. C
Sol.

Passage summary: The author is amazed that a French writer thinks that people are raising the question
of the notions of power only now. In fact, this whole notion has been a part of experience. Two pathologies
of power Stalinism and Fascism are apt examples. They did not do anything new but just extended and
used mechanisms already present in our societies. The role of philosophy is not just to keep reason within
the limit of experience but also to keep a check on the excessive powers of political rationality. The relation
between rationalisation and excesses of political power is evident. We don’t need to be reminded of it. We
also cannot use reason to examine and understand this relation. A better way to examine this relation is
not to take as a whole the rationalization of society or of culture but to analyze such a process in several
elds, each with reference to a fundamental experience: madness, illness, death, crime, sexuality, etc.

Genre: Philosophy

Number of words: 451


www.byjusexamprep.com

Type of question: Specific idea-based question

The two roles of philosophy are as follows:

1. (from the time of Kant) To prevent reason from going beyond the limits of what is given in experience

2. To keep watch over the excessive powers of political rationality, which is a rather high expectation

Point 2 is a different idea that the author expresses while explaining Stalinism and Fascism and not about
the role of philosophy. Point 1 looks like reason no. 1. But it is slightly different because of the word
unattainable. The passage just says that the job is a difficult one. This makes option C the answer.
23. D
Sol. Passage summary: Dharma has been misused by many people right from ancient times. It does not mean
religions or sects. It means the true universal eternal nature of all things. In the context of the Indian
constitution, the word secular has been wrongly translated as Dharma Nirpeksh. This is wrong as Dharma
is the truth, and governments will have to heed the truth. The teachings of the Buddha are not sectarian
but based on the quest for truth.

Genre: Religious philosophy

Number of words: 362

Type of question: Inference-based question

The answer to this question can be inferred from this sentence: ‘How can any government be non-
Dharmic? It has to be Dharmic, meaning it has to give importance to right conduct.’ Option D is just a
clever way of saying the same thing (that a government does not have the option of not following right
conduct.)

Options A and B are beyond the scope of the passage.

Option D: Following the law is the catch here. It may be right or perceived to be right. But following the law
is an extraneous idea.

Option E: The moment we use the word religious, the meaning changes. In the passage, religion is treated
differently from Dharma, which means what is universally right.
24. D
www.byjusexamprep.com

Sol.

Passage summary: Dharma has been misused by many people right from ancient times. It does not mean
religions or sects. It means the true universal eternal nature of all things. In the context of the Indian
constitution, the word secular has been wrongly translated as Dharma Nirpeksh. This is wrong as Dharma
is the truth, and governments will have to heed the truth. The teachings of the Buddha are not sectarian
but based on the quest for truth.

Genre: Religious philosophy

Number of words: 362

Type of question: Inference-based question

Dharma has been defined as the universal, eternal, and true nature of anything/anybody. Religion or sect
is not Dharma. This makes option D the answer.

Option A: the teachings of the Buddha are based on truth (and therefore Dharma), and following them is
not the same as following a religion or a sect according to the passage.

Option B: this is similar to what is described in the second paragraph as developing ‘negativity such as
aversion, jealousy, and animosity’ and these are examples of the negative form of Dharma.

Options C and E are very similar to the examples of Dharma (like fire and ice), which is the universal
nature of things.
25. A
Sol.

Passage summary: Dharma has been misused by many people right from ancient times. It does not mean
religions or sects. It means the true universal eternal nature of all things. In the context of the Indian
constitution, the word secular has been wrongly translated as Dharma Nirpeksh. This is wrong as Dharma
is the truth, and governments will have to heed the truth. The teachings of the Buddha are not sectarian
but based on the quest for truth.

Genre: Religious philosophy

Number of words: 362


www.byjusexamprep.com

Type of question: Inference-based question

Point i is against the fact that there are still instances where Dharam is used in the right sense. Refer to
these lines: ‘Dharma meant the universal laws of nature, which are applicable to everyone. Even today, we
come across Dharma being used in this sense.’ Point ii is in sync with the claim in the passage that the
word secular has been wrongly translated as Dharma Nirpeksh. It may have been wrongly translated but
the fact remains that it has been translated. Point iii is in line with what is clearly mentioned in the first
para: ‘Dharma is not Buddhist, Christian, Hindu, Jain, Jewish, Muslim, etc. These are all different religions
or sects.’ All of these reasons make A the answer.
26. E
Sol. i should be ‘Toxic waste should be handled and disposed of carefully.’
iii should be ‘He did not apprise me of the recent developments.’

v should be ‘Please fill in/out this form at the earliest.’

27. C
Sol. Whatever Roki has told Harish contains mostly the perception of one side and nothing concrete/actionable
to initiate disciplinary action. Moreover, the other side also needs to be heard. The best person for this job
is the HR manager. This means option C is the best answer. Option E is an attempt to wash one’s hands
off the problem.
28. A
Sol. Option A contradicts the claims made by the three. Option B does not render the claims invalid as what is
stated is the opinion of the three senior managers about what should have happened to their career and
not about the efficacy of Roki. Option C is a different issue. Moreover, a lack of evidence might mean that
he was innocent. Options D and E are also not pertinent to the issue.
29. C
Sol. Option A may have nothing to do with the skill of the three as it could be because Roki had everything
planned out for that period or other employees could have stepped up. Options B and D are opinions.
Option C is the option that comes closest to being evidence of the performance of these three people.
Option E is not enough as absence of complaints may not mean excellence.
30. D
Sol. The fact that the HR Head chose (or even thought of Ramesh) over others indicates that she has Ramesh
on her mind a week before the recommendations are going to be made. It is also given that this is the first
www.byjusexamprep.com

time she has personally emailed Ramesh. All of these put together should make Ramesh hopeful about
his promotion. Options A and C are contrary to what we are looking for. Options B and E suggest unethical
behaviour on the part of the HR Head and can be eliminated.
31. E
Sol. The only thing that Ramesh can do is follow up about what he was requested to do. It is completely
normal if he wants to know if the task has met its logical end as he was involved in it. This makes option E
the answer. Options A, B, and D would be taken as obtrusive and unwanted approaches as the promotion
process should not be revealed to anyone before the appraisal. Option C is a less viable option than
option E as it is still focused on the issue of the expected promotion.
32. B
Sol. The reason given by the HR Head is the mediocre performance of Ramesh, and it cannot be refuted
without any concrete evidence (which is not given or indicated in the information given). This makes option
B the best answer.
33. D
Sol. The most feasible answer among the options is option D. What RKS is receiving online is based on issues
that are not connected to the main problem at all. This could be a case of meaningless online action over
which nobody has any control. Option A is out of the question as there does not seem to be any crime
involved and we cannot expect the authorities to take actions against the detractors of a star without any
solid basis. Option B borders on being illegal. Options C and E are too extreme steps.
34. C
Sol. The fact of the matter is that RKS has received a summons from a court, and it cannot be neglected.
Option C provides the additional safety net of the concerned authority’s exoneration. This is the best way
out.
35. D
Sol. The TV-programme claims that there is arsenic in the groundwater. It does not say anything about arsenic
in POLA’s products. It is also given in the question that an enquiry is likely to take place. Option D provides
the best way out. Options A, C, and E assume that arsenic is present in POLA’s products. This is
something that we do not know yet. Option B misses the point as POLA would be in trouble if arsenic is
found in its products.
36. A
Sol. In this situation, not doing anything is the best out of the given solutions because it could just be a rumour.
Hence, option E is the correct answer.
www.byjusexamprep.com

While B talks about not doing anything, the reason given behind it is not sound. As Jabir had helped in
resolving the performance concerns earlier, those concerns may no longer exist.

C can lead to spreading of the rumour and even a snowballing effect and may have negative
consequences for Mayank and/or the organisation.

D is an extreme solution because Jabir is not even sure whether Mayank intends to resign.

E can make Mayank’s manager prejudiced towards him for no reason, and is an unsound solution.
37. D
Sol. Option A seems to be in the nature of a punitive measure since Jabir’s judgement of Mayank’s
expressions and tones could be flawed. Moreover, the prospect of a bond can lead to Mayank actually
resigning even if he were not planning to do so.

Option B is again unsound as it might negatively affect the morale of the employee and penalises him for
something that the HR Manager is merely unsure about.

Option C is unsound because ‘commitment toward the organisation’ is a broad term that encompasses
Mayank’s effectiveness, efficiency, initiative in his work, etc. So, giving him a low rating based on just a
hunch is disproportionate.

Option D is apt as this ensures that he is fairly treated for his past performance. It also follows a wait-and-
watch approach as far as other actions, let’s say, promoting him is concerned.

Option E is akin to promoting office gossip and rumours and amounts to going behind Mayank’s back.
38. E
Sol. Clearly, option E is the most balanced recommendation given the multiple angles involved in the matter.

It is possible that Mayank wanted to move to Bengaluru, but his resignation has still been prompted by
discrimination. So, option A is not a sound recommendation.

Option B assumes that discrimination has happened and is speculating the reasons behind it.

Option C is again assuming too much and speculating the reasons.

Option D is far-fetched as Jabir’s enquiry has not revealed any explicit discrimination.
www.byjusexamprep.com

39. A
Sol. Option E looks tempting, but it’s unethical to share the assessment results and details with anyone. Option
B is curt and will not satisfy Ramesh. Further, in option B, the question of contacting the HR director is
inappropriate. Option C does not provide any solution at all. Option D is a threat and may force Ramesh to
retaliate and the issue can get out of hand. Option A is correct as it’s important to explain the nitty gritties
of the hiring process with the candidates.
40. C
Sol. i should be the first step as the first step is to ascertain the budget. Hence, we are left with options C and
E. ii should be the last statement as sending emails regarding the schedule should be the last thing.
Hence, the correct answer is C.
41. C
Sol. Step ii is not all to be included as it has nothing to do with the filling up of the vacancies. In fact, it would
be inappropriate to expect a quid pro quo from the Director. This leaves us with options C and D. Of these,
C is the best as the first thing to be done is to get the selection panel in place today which is mentioned in
i. The next step is to conduct the onsite pre interview test mentioned in iii, and then group discussions will
be conducted which is mentioned in iv. Note that ‘initial applicants’ is important in iii.
42. E
Sol. Step iv is to be eliminated as it is not at all practical to organise this session as it may involve teachers
from other organisations too. This leaves us with options B and E. E is the best answer as the decision of
retaining teachers may have to be taken only after meeting the representatives.
43. B
Sol. The key here is to satisfy all the stakeholders, which means the management as well as the teachers.
Options A and E are out of the question as the issue with the teachers is not monetary. Option C is not
practical as one trainer may not be equipped to train teachers from many fields. Option D does not
consider the perspective of the teachers at all.
44. D
Sol. Option A is not suitable as it conveys that you are not willing to take up the new challenge. Option C is out
of the question as it looks like you are entering into an unethical nexus with the senior teacher. In Option
E, you are inviting trouble as the senior teacher may not agree with the arrangement and you are not
authorised to do anything about it. Between options B and D, D is more positive in approach. Moreover,
letting everyone know how things will be is quite vague.
45. C
www.byjusexamprep.com

Sol. The main concern here is to break the news at the earliest keeping in mind that the father had just had a
nervous breakdown. Option A is inappropriate as Abani is still recuperating. Option B does very little to
reduce the disappointment that is going to be caused to the father as his desire was to see his son in his
organisation. On the contrary, it might just worsen the situation. Option C presents the best opportunity for
Sunil to meet the father, and it would be best to break the news in person. Also, Abani has completely
recovered now. Option D is irrelevant as it is just what Sukhesh wants and has little to do with the father-
son relation dynamics between Abani and Sunil. Option E: a letter is not the best way to break this news.
46. E
Sol. Option A: the fact that Sridevi has been a housewife does not guarantee her proficiency in handling a new
plant. Option B: Asking Shikha to handle two plants at the same time while shuttling across the 50 km
distance will be an overburden on Shikha. Option C: It is just a stop-gap arrangement. Option D: It is not
clear in what capacity Clara works. Her skills and experience may not be just enough for being the head of
a plant. Option E: Ravi is the best choice. He has been with Abani Sons long enough to be trusted. He is
the deputy head of another plant and is the one most likely to be capable of handling a plant.
47. E
Sol. The apprehension here is whether Rekha would take sides with her father and try to influence her
brother’s decision. Option A is just an example of Rekha living her dream in her own way. Option B looks
close but this involves a social issue, and the employees are not family members also. Option C is more
about how Sunil would respond to any disagreements with his choice and not about Rekha. Option D does
not indicate that Rekha will defy her father. It is just a mention of a past incident with which she has
reconciled. Option E is the best choice as it demonstrates how Rekha can support her brother and not her
father.
48. D
Sol. By writing single-digit numbers from 1 to 9, we find that 9 × 1 = 9 digits are covered.

By writing two-digit numbers from 10 to 99, we find that 90 × 2 = 180 digits are covered.

A total of 189 digits are covered.

The remaining 1200 – 189 = 1011 digits will be covered by three-digit numbers.

To write 1011 digits, we require = 337 numbers.


www.byjusexamprep.com

By starting from 100, we find that the smallest three-digit number, i.e., the 337rd number will be
436.

Hence, option D is the correct answer.

49. C
Sol. Let the distance between points X and Y be ‘d’.
Let the speed of drivers starting from Point X and Point Y be s1 and s2, respectively.
So, when they met during their onward journey, the driver who started from Point Y covered 20
km. Then, the driver at Point X covered (d – 20) km.

So,
Distance covered by the driver who started from Point Y till he met the other driver the second
time = (d + 12) km
Distance covered by the other driver in the same time interval = (d + d – 12) = 2d – 12 km

So,
From (1) and (2), we get the following:

=
km
Hence, option C is the correct answer.

50. C
Sol. Sum of the number of students selected from courses A and B who are not athletes = (100 –
40)% of 20% of 400 + (100 – 55)% of 25% of 400 = 48 + 45 = 93

Sum of the number of students selected from courses C and D who are athletes = 25% of 15%
of 400 + 50% of 40% of 400 = 15 + 80 = 95

Required answer = × 100 = 97.89% ≈ 98%

Hence, option C is the correct answer.


www.byjusexamprep.com

51. B
Sol. Let N be the number that gives the remainder as 7, 6, 4, and 3 when divided by 9, 8, 6, and 5,
respectively.
The common difference between divisors and remainders is constant.
(9 – 7) = (8 – 6) = (6 – 4) = (5 – 3) = 2
N = LCM(divisors) × K – common difference
[K is a constant.]
N = LCM(9, 8, 6, 5) – common difference
LCM (9, 8, 6, 5) = 360
To find the greatest 4-digit number, divide 9999 with 360.

So, the greatest 4-digit number that is divisible by 360 is 9720.


The required answer is 9720 – 2 = 9718.
Hence, option B is the correct answer.
52. D
Sol. Let the marks obtained by him in the five tests be a, b, c, d, and e, where a < b < c < d < e.

By arranging the given averages in ascending order, we get the following:

77, 84, 89, 91, 103, 108, 110, 115, 117, and 122

Since a and b are the smallest, we get the following:

= 77 …(i)

Or a + b = 154

Similarly, = 122

Or d + e = 244 …(ii)

Sum of all the given averages = 77 + 84 + 89 + 91 + 103 + 108 + 110 + 115 + 117 + 122 =
1016
www.byjusexamprep.com

We are given a total of 10 averages, so it must have 20 numbers.

Thus, each number would have been counted = 4 times.

So, 4(a + b + c + d + e) = 1016 × 2

(Multiplied RHS by 2 because all the numbers are averages in the LHS)

Thus, a + b + c + d + e = 508

By using (i) and (ii), we get the following:

154 + c + 244 = 508

Or 110

Now, d + e = 244

Also, c < d < e.

So, the least possible value of d is 111. But we should have a number that is the average of c
and d, which is not there.

So, d ≠ 111

For d = 112, the average of c and d should be = 111 (which is not there in the given
numbers).

The least possible value of ‘d’ that can satisfy according to the given averages is 120, for

which the average of c and d should be = 115.

Thus, e = 124

This further confirms the values because = 117, which is one of the given averages.

So, d = 120

Hence, option D is the correct answer.


www.byjusexamprep.com

53. A
Sol. From the chart, we can draw the following table:
Let the names of the five laptops be coded as A, B, C, D, and E.

Let us calculate the performance rating (PR) and the overall rating (OR) for all the five laptops.

P refers to price, PC refers to processor, S refers to storage, and R refers to RAM.


Using statement 1:
Performance ratings of Luka and Gubera were the same, but Luka had a better overall rating
than Gubera.
Clearly, E is Luka, and B is Gubera.
Using statement 2:
Overall rating of Marty was less than Panto, but its performance rating was better than Jamiko.
Thus, D is Marty, C is Panto, and A is Jamiko.

He would buy Plato since it has the highest final rating.


Hence, option A is the correct answer.
www.byjusexamprep.com

54. A
Sol. From the chart, we can draw the following table:
Let the names of the five laptops be coded as A, B, C, D, and E.

Let us calculate the performance rating (PR) and the overall rating (OR) for all the five laptops.

P refers to price, PC refers to processor, S refers to storage, and R refers to RAM.


Using statement 1:
Performance ratings of Luka and Gubera were the same, but Luka had a better overall rating
than Gubera.
Clearly, E is Luka, and B is Gubera.
Using statement 2:
Overall rating of Marty was less than Panto, but its performance rating was better than Jamiko.
Thus, D is Marty, C is Panto, and A is Jamiko.

From the final table, Gubera has an overall rating of 5.


55. C
www.byjusexamprep.com

Sol. From the chart, we can draw the following table:


Let the names of the five laptops be coded as A, B, C, D, and E.

Let us calculate the performance rating (PR) and the overall rating (OR) for all the five laptops.

P refers to price, PC refers to processor, S refers to storage, and R refers to RAM.


Using statement 1:
Performance ratings of Luka and Gubera were the same, but Luka had a better overall rating
than Gubera.
Clearly, E is Luka, and B is Gubera.
Using statement 2:
Overall rating of Marty was less than Panto, but its performance rating was better than Jamiko.
Thus, D is Marty, C is Panto, and A is Jamiko.

The absolute difference between the final rating of Gubera and Panto is 102 – 81 = 21.
Hence, option C is the correct answer.
56. D
www.byjusexamprep.com

Sol. Put y = 7 in .

or

Hence, option D is the correct answer.


57. E
Sol. The number of odd days till 31 December 2000 is zero, and 31 December 2000 is a Sunday.
From 1 January 2001 till 31 December 2012, there are 3 leap years and 9 normal years.
Hence, (which is one odd day)
From 1 January 2013 till 1 February 2013, there are 32 days or 4 days.
The total number of odd days is 5. Hence, 1 February 2013 is a Friday.
Hence, option E is the correct answer.
58. A
Sol.
p= = 30 + 31 + 32 + 33 + … + 372 + 373

We need to find the remainder when p is divided by 11.

Now, 30 + 31 + 32 + 33 + 34 = 1 + 3 + 9 + 27 + 81 = 121

It is completely divisible by 11.

So, we will have the following:

(30 + 31 + 32 + 33 + 34) + 35(30 + 31 + 32 + 33 + 34) + …… + 365(30 + 31 + 32 + 33 + 34) +


370(30 + 31 + 32 + 33).
www.byjusexamprep.com

Now, (30 + 31 + 32 + 33 + 34) + 35(30 + 31 + 32 + 33 + 34) + …… + 365(30 + 31 + 32 + 33 + 34)


will be divisible by 11.

We need to find the remainder when 370(30 + 31 + 32 + 33) or 370(40) is divided by 11.

370 = (35)14 = (243)14 = (242 + 1)14

So, the remainder when 370 is divided by 11 is 1.

Remainder when 40 is divided by 11 = 7

Required remainder = 1 × 7 = 7

Hence, option A is the correct answer.

59. C
Sol. If we look at the sides of Triangle ABC, we see that they form a Pythagorean triplet, i.e., (10, 24, 26).

So, ∠BAC = 90°

Also, G is the centroid (which lies on Side AE)

This means that AE is the median.

BE = EC

If we draw a circle that passes through points A, B, and C, then AE, BE, and EC are its radius.

So, AE = BE = EC = 13 cm

As G is the centroid, it will divide Side AE in the ratio 2 : 1.

Thus, GE = cm

Hence, option C is the correct answer.


www.byjusexamprep.com

60. C
Sol.

Here, is satisfied from 68 or 128.


To get , the minimum value of P must be .
Hence, option C is the correct answer.
61. B
Sol. In group B, the maximum difference between the points scored by any two teams is 24. Thus,
one team has won all the games and one team has lost all the games. Therefore, the scores
for group B in the first round will be –12, –4, +4, and +12.
In group A, the maximum difference between the points scored by any two teams is 8. The
points scored will be –4, –4, 4, and 4.
After the second round, the maximum difference between the points scored by any two teams
is 20.
Now, the teams that scored 12 and 4 points after the first round will qualify for the second
round from group B. The teams that scored 4 points each will qualify for the second round
from group A.
Clearly, the team that has 12 points (group B) must have beaten the team having 4 points
(group A) in the second round, so the winner moves to 18 points and the loser moves to –2
points, while in the other second round match, both the teams are at +4, so the team from
group A will beat the team of group B, so the winner moves to 10 and the loser to –2.
Thus, the points will be 18, 10, –2, and –2.
Clearly, from the common solution, the team from group A who played the finals had scored
10 points at the end of the second round of the tournament.
Hence, option B is the correct answer.
62. C
Sol. In group B, the maximum difference between the points scored by any two teams is 24. Thus,
one team has won all the games and one team has lost all the games. Therefore, the scores
www.byjusexamprep.com

for group B in the first round will be –12, –4, +4, and +12.
In group A, the maximum difference between the points scored by any two teams is 8. The
points scored will be –4, –4, 4, and 4.
After the second round, the maximum difference between the points scored by any two teams
is 20.
Now, the teams that scored 12 and 4 points after the first round will qualify for the second
round from group B. The teams that scored 4 points each will qualify for the second round
from group A.
Clearly, the team that has 12 points (group B) must have beaten the team having 4 points
(group A) in the second round, so the winner moves to 18 points and the loser moves to –2
points, while in the other second round match, both the teams are at +4, so the team from
group A will beat the team of group B, so the winner moves to 10 and the loser to –2.
Thus, the points will be 18, 10, –2, and –2.
If Akka has won only against Nakka, then Akka has –4 points. Also, Dakka has lost only one
match, and he is at +4. Since Nakka has lost one match as well, Takka is the team who has
won all the matches in the first round. Thus, Takka would have won the tournament.
63. C
Sol. In group B, the maximum difference between the points scored by any two teams is 24. Thus,
one team has won all the games and one team has lost all the games. Therefore, the scores
for group B in the first round will be –12, –4, +4, and +12.
In group A, the maximum difference between the points scored by any two teams is 8. The
points scored will be –4, –4, 4, and 4.
After the second round, the maximum difference between the points scored by any two teams
is 20.
Now, the teams that scored 12 and 4 points after the first round will qualify for the second
round from group B. The teams that scored 4 points each will qualify for the second round
from group A.
Clearly, the team that has 12 points (group B) must have beaten the team having 4 points
(group A) in the second round, so the winner moves to 18 points and the loser moves to –2
www.byjusexamprep.com

points, while in the other second round match, both the teams are at +4, so the team from
group A will beat the team of group B, so the winner moves to 10 and the loser to –2.
Thus, the points will be 18, 10, –2, and –2.
Chakka won three matches in the tournament: 2 of the first round and 1 of the second round.
Thus, he lost two matches, one in the first round and one in the finals.
Net points: +4 (first round) + 6 (second rounds) – 8(finals) = +2 points
Bakka lost only to Chakka in the first round. Thus, he had +4 points in the first round and lost
in the second round (–6).
Net points: +4 (first round) – 6(second round) = –2
The absolute difference between the two points of Chakka and Bakka is 4.
Hence, option C is the correct answer.
64. C
Sol. Total ways to select 3 squares in a chessboard

Total ways in which 3 squares are selected such that no two squares are not on the same row

or same column

Required probability
Hence, option C is the correct answer.
65. C
Sol.
It is given that each vowel represents an odd number, and each consonant represents an even
number.

Option A: b2 will be even, a3 will be odd, 3be2 will be even, and xy will be even.

So, b2 + a3 – 3be2 + xy = even + odd – even + even = odd

So, option A is false.

Similarly, when we check for other options, we can see that only option C is true.
www.byjusexamprep.com

Hence, option C is the correct answer.

66. C
Sol. Let the number of movies shown on screens 2, 3, 4, and 7 be x, and, and the number of movies shown on
screens 1 and 6 be y; the number of movies shown on Screen 5 be z.

4x + 2y + z = 4 5 = 20 …(1)

x + y + z = 9 …(2)

By subtracting (2) from (1), we get the following:

3x + y = 11

The only possible solution is x = 3 and y = 2.

So, by solving for z, we get z = 4.

As the number of movies shown on Screen 5 is 4, the movies except Predator were shown on Screen 5.

By using statements 1 , we get the following:


www.byjusexamprep.com

By using statements 2 , we get the following:

Predator and Gladiator must have been shown on Screen 4.

Gladiator must have been shown on screens 5 and 7.

Predator and Lethal Weapon must have been shown on Screen 3.

By using statements 3 , we get the following:


www.byjusexamprep.com

Now, Gladiator cannot be shown on screen 6 as it violates the conditions.

The Bourne Identity was definitely shown on screen 2 and screen 5.


67. C
Sol. Let the number of movies shown on screens 2, 3, 4, and 7 be x, and, and the number of movies shown on
screens 1 and 6 be y; the number of movies shown on Screen 5 be z.

4x + 2y + z = 4 5 = 20 …(1)

x + y + z = 9 …(2)

By subtracting (2) from (1), we get the following:

3x + y = 11

The only possible solution is x = 3 and y = 2.

So, by solving for z, we get z = 4.

As the number of movies shown on Screen 5 is 4, the movies except Predator were shown on Screen 5.
www.byjusexamprep.com

By using statements 1 , we get the following:

By using statements 2 , we get the following:

Predator and Gladiator must have been shown on Screen 4.

Gladiator must have been shown on screens 5 and 7.

Predator and Lethal Weapon must have been shown on Screen 3.


www.byjusexamprep.com

By using statements 3 , we get the following:

Now, Gladiator cannot be shown on screen 6 as it violates the conditions.

Statements (i), (iii) and (iv) are definitely correct, while the other two statements might be correct or
incorrect.
68. C
Sol. Let the number of movies shown on screens 2, 3, 4, and 7 be x, and, and the number of movies shown on
www.byjusexamprep.com

screens 1 and 6 be y; the number of movies shown on Screen 5 be z.

4x + 2y + z = 4 5 = 20 …(1)

x + y + z = 9 …(2)

By subtracting (2) from (1), we get the following:

3x + y = 11

The only possible solution is x = 3 and y = 2.

So, by solving for z, we get z = 4.

As the number of movies shown on Screen 5 is 4, the movies except Predator were shown on Screen 5.

By using statements 1 , we get the following:


www.byjusexamprep.com

By using statements 2 , we get the following:

Predator and Gladiator must have been shown on Screen 4.

Gladiator must have been shown on screens 5 and 7.

Predator and Lethal Weapon must have been shown on Screen 3.

By using statements 3 , we get the following:


www.byjusexamprep.com

Now, Gladiator cannot be shown on screen 6 as it violates the conditions.

Only for Predator, Lethal Weapon and Gladiator the four screen numbers can be uniquely determined.
69. E
Sol.
Let the two numbers be 32a and 32b, respectively (where ‘a’ and ‘b’ are coprime numbers).

LCM = 32ab = 576

So, ab = 18 = 22 × 3

The possible values of (a, b) are (1, 18) and (2, 9).

So, the two numbers are (32, 576) or (64, 288).

Required answer = 32 + 64 = 96

Hence, option E is the correct answer.

70. C
Sol. Let P and Q be the centres of the big circles and R be the centre of the small circle.
www.byjusexamprep.com

Let S, T, and U be their point of contact as shown in the figure.

Length of PQ = 3 + 3 +3+3 =6+6 cm

Length of PR = Length of QR = 3 + 3 +3=6+3 cm

From the given values, we can say that .

∠PRQ = 90°

So, △PRQ is an isosceles right-angled triangle.

Thus, ∠RPQ = ∠RQP = 45°

So, Arc length TU =

Arc length ST = SU =

So, the required perimeter = + cm


www.byjusexamprep.com

Hence, option C is the correct answer.


71. E
Sol. From statement I, we get the following:

Let the number of coins of 25 paise, 50 paise, and Rs. 1 be 3k, 4k, and 5k, respectively.
From this data, we cannot find the number of individual coins.
Thus, statement I alone is not sufficient.

From statement II, we get the following:

Statement II alone is also not sufficient to answer the question because it provides only the
ratio of difference between the values and not the actual number of coins.

Using statement I and II together, we can write the following:

,
We cannot find the number of individual coins.
Hence, option E is the correct answer.
72. B
Sol. From the data given in the chart, we can make the following table:

Piyush’s attendance is equal to half of Sudha’s attendance for two different sessions on
exactly two occasions. Also, Piyush’s attendance is double of Amrit’s attendance in two
different sessions on exactly one occasion.
Amiya’s attendance and Sudha’s attendance are equal in one session, which in turn is double
of Piyush’s attendance for one session and it happened in three different sessions for these
www.byjusexamprep.com

three people.

The attendance of Som is more than 80% (more than 40 out of 50 working days) in sessions 3
and 4.
73. E
Sol. From the data given in the chart, we can make the following table:

Piyush’s attendance is equal to half of Sudha’s attendance for two different sessions on
exactly two occasions. Also, Piyush’s attendance is double of Amrit’s attendance in two
different sessions on exactly one occasion.
Amiya’s attendance and Sudha’s attendance are equal in one session, which in turn is double
of Piyush’s attendance for one session and it happened in three different sessions for these
three people.

The attendances of Amiya in sessions 1 and 2 are 30 and 40, which are multiples of 5.
74. E
Sol. From the data given in the chart, we can make the following table:
www.byjusexamprep.com

Piyush’s attendance is equal to half of Sudha’s attendance for two different sessions on
exactly two occasions. Also, Piyush’s attendance is double of Amrit’s attendance in two
different sessions on exactly one occasion.
Amiya’s attendance and Sudha’s attendance are equal in one session, which in turn is double
of Piyush’s attendance for one session and it happened in three different sessions for these
three people.

The attendance of Amrit was never ranked 2nd or 3rd.


75. E
Sol.

As
[The denominator is 0 at which is not allowed.]

Let us try to find the value of .


www.byjusexamprep.com

The critical points of are 15 and 6.


For , say 100,
The value of is either zero or positive.

is always positive.
So, is always zero or positive.
Hence no solution for this range.
For , say 8,
The value of is always negative.

But the value of is always positive when the value of x is 14.


For , say 1,
The value of is either zero or positive.

But the value of is always negative when the values of x are 5 or 4.


The number of integral solutions is 3, and these values are 4, 5 and 14.
Hence, option E is the correct answer.

You might also like